Real Numbers

You might also like

Download as pdf or txt
Download as pdf or txt
You are on page 1of 52

STUDY

PACKAGE

REAL NUMBERS

My Home Jewel, Madinaguda,

Hyderabad-500049
Std. 10th |MATHEMATICS

Chapter

1
REAL NUMBERS

Introduction

Euclid’s division algorithm, as the name suggests, has to do with divisibility of integers. Stated
simply, it says any positive integer a can be divided by another positive integer b in such a way that
it leaves a remainder r that is smaller than b. Many of you probably recognise this as the usual long
division process. Although this result is quite easy to state and understand, it has many applications
related to the divisibility properties of integers. We touch upon a few of them, and use it mainly to
compute the HCF of two positive integers.
The Fundamental Theorem of Arithmetic, on the other hand, has to do something with
multiplication of positive integers. You already know that every composite number can be expressed
as a product of primes in a unique way—this important fact is the Fundamental Theorem of
Arithmetic. Again, while it is a result that is easy to state and understand, it has some very deep and
significant applications in the field of mathematics. We use the Fundamental Theorem of Arithmetic
for two main applications. First, we use it to prove the irrationality of many of the numbers you
studied in Class IX, such as 2, 3 and 5 . Second, we apply this theorem to explore when exactly
p
the decimal expansion of a rational number, say  q  0  , is terminating and when it is non-
q
p
terminating repeating. We do so by looking at the prime factorisation of the denominator q of .
q
You will see that the prime factorisation of q will completely reveal the nature of the decimal
p
expansion of .
q
So let us begin our exploration.

www.mstclasses.com, www.satpinnacle.com +91-9140474462, +91-8106429979 |1


Std. 10th |MATHEMATICS

Euclid’s Division Lemma


Consider the following folk puzzle
A trader was moving along a road selling eggs. An idler who didn’t have much work to do,
started to get the trader into a wordy duel. This grew into a fight, he pulled the basket with eggs and
dashed it on the floor. The eggs broke. The trader requested the Panchayat to ask the idler to pay for
the broken eggs. The Panchayat asked the trader how many eggs were broken. He gave the following
response:
If counted in pairs, one will remain;
If counted in threes, two will remain;
If counted in fours, three will remain;
If counted in fives, four will remain;
If counted in sixes, five will remain;
If counted in sevens, nothing will remain;
My basket cannot accommodate more than 150 eggs.
So, how many eggs were there? Let us try and solve the puzzle. Let the number of eggs be a. Then
working backwards, we see that a is less than or equal to 150:
If counted in sevens, nothing will remain, which translates to a = 7p + 0, for
some natural number p. If counted in sixes, a = 6q + 5, for some natural number q.
If counted in fives, four will remain. It translates to a = 5w + 4, for some natural
number w.
If counted in fours, three will remain. It translates to a = 4s + 3, for some natural
number s.
If counted in threes, two will remain. It translates to a = 3t + 2, for some natural
number t.
If counted in pairs, one will remain. It translates to a = 2u + 1, for some natural
number u.
That is, in each case, we have a and a positive integer b (in our example,

www.mstclasses.com, www.satpinnacle.com +91-9140474462, +91-8106429979 |2


Std. 10th |MATHEMATICS

b takes values 7, 6, 5, 4, 3 and 2, respectively) which divides a and leaves a remainder


r (in our case, r is 0, 5, 4, 3, 2 and 1, respectively), that is smaller than b.
This is modified form of a puzzle given in ‘Numeracy Counts!’ by A. Rampal, and others.
moment we write down such equations we are using Euclid’s division lemma, which is given in
Theorem 1.1.
Getting back to our puzzle, do you have any idea how you will solve it? Yes! You must look
for the multiples of 7 which satisfy all the conditions. By trial and error (using the concept of LCM),
you will find he had 119 eggs.
In order to get a feel for what Euclid’s division lemma is, consider the following pairs of
integers:
17, 6; 5, 12; 20, 4
Like we did in the example, we can write the following relations for each such pair:
17 = 6 × 2 + 5 (6 goes into 17 twice and leaves a remainder 5)

5 = 12 × 0 + 5 (This relation holds since 12 is larger than 5)

20 = 4 × 5 + 0 (Here 4 goes into 20 five-times and leaves no remainder)


That is, for each pair of positive integers a and b, we have found whole numbers q and r,
satisfying the relation:
a  bq  r, 0  r  b

Note that q or r can also be zero.


Why don’t you now try finding integers q and r for the following pairs of positive integers a and b?
(i) 10, 3; (ii) 4, 19; (iii) 81, 3
Did you notice that q and r are unique? These are the only integers satisfying the conditions
a = bq + r, where 0  r  b . You may have also realised that this is nothing but a restatement of the
long division process you have been doing all these years, and that the integers q and r are called the
quotient and remainder, respectively.
A formal statement of this result is as follows:
Euclid’s Division Algorithm
Given positive integers a and b, there exist unique integers q and r satisfying a = bq + r,
0  r  b.

www.mstclasses.com, www.satpinnacle.com +91-9140474462, +91-8106429979 |3


Std. 10th |MATHEMATICS

This result was perhaps known for a long time, but was first recorded in Book VII of Euclid’s
Elements. Euclid’s division algorithm is based on this lemma.
Euclid’s division algorithm is a technique to compute the Highest Common Factor (HCF) of two
given positive integers. Recall that the HCF of two positive integers a and b is the largest positive
integer d that divides both a and b.
Let us see how the algorithm works, through an example first. Suppose we need to find the
HCF of the integers 455 and 42. We start with the larger integer, that is, 455. Then we use Euclid’s
lemma to get
455  42 10  35
Now consider the divisor 42 and the remainder 35, and apply the division lemma to get
42  35 1  7
Now consider the divisor 35 and the remainder 7, and apply the division lemma to get
35  7  5  0
Notice that the remainder has become zero, and we cannot proceed any further.
We claim that the HCF of 455 and 42 is the divisor at this stage, i.e., 7. You can easily verify this by
listing all the factors of 455 and 42. Why does this method work? It works because of the following
result.
So, let us state Euclid’s division algorithm clearly.
To obtain the HCF of two positive integers, say c and d, with c > d, follow the steps below:
Step 1: Apply Euclid’s division lemma, to c and d. So, we find whole numbers, q and r such
that c  dq  r, 0  r  d .

Step 2: If r = 0, d is the HCF of c and d. If r  0 , apply the division lemma to d and r.


Step 3: Continue the process till the remainder is zero. The divisor at this stage will be the
required HCF.

This algorithm works because HCF (c, d) = HCF (d, r) where the symbol HCF (c, d) denotes the HCF
of c and d, etc.
Question 1:
Use Euclid’s algorithm to find the HCF of 4052 and 12576.
Sol:
Step 1: Since 12576 > 4052, we apply the division lemma to 12576 and 4052, to get
12576  4052  3  420

www.mstclasses.com, www.satpinnacle.com +91-9140474462, +91-8106429979 |4


Std. 10th |MATHEMATICS

Step 2: Since the remainder 420  0, we apply the division lemma to 4052 and 420, to get
4052  420  9  272
Step 3: We consider the new divisor 420 and the new remainder 272, and apply the division
lemma to get
420  272 1  148
We consider the new divisor 272 and the new remainder 148, and apply the division lemma to get

272  148 1  124


We consider the new divisor 148 and the new remainder 124, and apply the division lemma to get

148  124 1  24
We consider the new divisor 124 and the new remainder 24, and apply the division lemma to get

124  24  5  4
We consider the new divisor 24 and the new remainder 4, and apply the division lemma to get

24  4  6  0
The remainder has now become zero, so our procedure stops. Since the divisor at this stage is 4, the
HCF of 12576 and 4052 is 4.
Notice that 4 = HCF (24, 4) = HCF (124, 24) = HCF (148, 124) = HCF (272, 148) = HCF (420, 272)
= HCF (4052, 420) = HCF (12576, 4052).
Euclid’s division algorithm is not only useful for calculating the HCF of very large numbers,
but also because it is one of the earliest examples of an algorithm that a computer had been
programmed to carry out.

Question 2:

Obtain HCF of 12056 and 420.


Sol: Step 1: Since 12056 > 420, we apply division lemma to 12056 and 420 to get

12056  420  28  296 .


Since the remainder 296  0 , we apply the division lemma to 420 and 296.

Step 2: 420  296 1  124 .


Again, remainder is not zero.
 division lemma is again applied to 296 and 124.
www.mstclasses.com, www.satpinnacle.com +91-9140474462, +91-8106429979 |5
Std. 10th |MATHEMATICS

Step 3: 296  124  2  48 . Now, 48 becomes the new divisor and 124 the new dividend.
Step 4: 124  48  2  28
Step 5: 48  28 1  20
Step 6:` 28  20 1  8
Step 7: 20  8  2  4
Step 8: 8  4 2  0
Here, the remainder becomes zero, so we stop the algorithm here. Since, the divisor at this stage is 4,
HCF of 12056 and 420 is 4.
Notice that 4 = HCF (8, 4) = HCF (20, 8) = HCF (28, 20)
= HCF (48, 28) = HCF (124, 48)
= HCF (296, 124) = HCF (420, 296) = HCF (12056, 420)

Question 3:

Show that any positive odd integer is of the form of 8q + 1 or 8q + 3 or 8q + 5 or 8q + 7, where q is


some integer.
Sol: Let ‘a’ be any positive odd integer and b = 8. If we apply Euclid’s division lemma, r can be
0, 1, 2, 3, 4, 5, 6 or 7.
i.e. a can be 8q, 8q + 1, 8q + 2, 8q + 3, 8q + 4, 8q + 5, 8q +6, 8q + 7.
Since a is odd, it cannot be of the form 8q, 8q + 2, 8q + 4, 8q + 6.
 any positive odd integer will be of the form of 8q + 1, 8q + 3, 8q + 5 or 8q + 7.

Question 4:

Show that every positive even integer is of the form 2q, and that every positive odd integer is of the
form 2q + 1, where q is some integer.
Sol: Let a be any positive integer and b = 2. Then, by Euclid’s algorithm, a = 2q + r, for some integer
q  0 , and r = 0 or r = 1, because 0  r  2 . So, a = 2q or 2q + 1.

If a is of the form 2q, then a is an even integer. Also, a positive integer can be either even or odd.
Therefore, any positive odd integer is of the form 2q + 1.

www.mstclasses.com, www.satpinnacle.com +91-9140474462, +91-8106429979 |6


Std. 10th |MATHEMATICS

Question 5:

Show that any positive odd integer is of the form 4q + 1 or 4q + 3, where q is some integer.
Sol: Let us start with taking a, where a is a positive odd integer. We apply the division algorithm with a
and b = 4.

Since 0  r  4 , the possible remainders are 0, 1, 2 and 3.


That is, a can be 4q, or 4q + 1, or 4q + 2, or 4q + 3, where q is the quotient.
However, since a is odd, a cannot be 4q or 4q + 2 (since they are both divisible by 2).
Therefore, any odd integer is of the form 4q + 1 or 4q + 3.
Question 6:

A sweet seller has 420 kaju barfis and 130 badam barfis. She wants to stack them in such a way that
each stack has the same number, and they take up the least area of the tray. What is the number of
that can be placed in each stack for this purpose?
Sol: This can be done by trial and error. But to do it systematically, we find HCF (420, 130).
Then this number will give the maximum number of barfis in each stack and the number of stacks
will then be the least. The area of the tray that is used up will be the least.
Now, let us use Euclid’s algorithm to find their HCF. We have:

420  130  3  30
130  30  4  10
30  10  3  0
So, the HCF of 420 and 130 is 10. Therefore, the sweet seller can make stacks of 10 for both kinds of
barfi.

www.mstclasses.com, www.satpinnacle.com +91-9140474462, +91-8106429979 |7


Std. 10th |MATHEMATICS

Exercise 1:

1. Show that cube of any positive integer is of the form of 9m, 9m + 1 or 9m + 8.


2. Use Euclid’s division algorithm to find HCF of 460 and 20468.

3. Shat that n 2  1 is divisible by 8, if n is an odd positive integer.


4. Prove that product of every three consecutive integers is always divisible by 3 as well as 2.
5. Use Euclid’s division algorithm to find the HCF of:
(i) 135 and 225 (ii) 196 and 38220 (iii) 867 and 255
6. Show that any positive odd integer is of the form 6q + 1, or 6q + 3, or 6q + 5, where q is some integer.
7. An army contingent of 616 members is to march behind an army band of 32 members in a parade.
The two groups are to march in the same number of columns. What is the maximum number of
columns in which they can march?
8. Use Euclid’s division lemma to show that the square of any positive integer is either of the form 3m
or 3m + 1 for some integer m.
9. Use Euclid’s division lemma to show that the cube of any positive integer is of the form 9m, 9m + 1
or 9m + 8.

The fundamental Theorem of Arithmetic


The theorem states that every composite number can be expressed as a product of prime numbers,
and their factorisation is unique, apart from the order in which the prime factors occur.
i.e. given any composite number, there is one and only one way to write it as a product of primes.

Question 1:

Consider the numbers of the form 3n (where n s a natural number). Check whether there is any value
of n for which 3n ends with 2.

Sol: If a number of the form 3n ends with 2, then it will be divisible by 2. i.e. the prime number 2. This is
not possible because prime factorisation of 3n contains only 3. The uniqueness property of the
fundamental theorem of Arithmetic guarantees that there are no other primes in the factorisation of
3n . Hence, there is no natural number n for which 3n ends with 2.

www.mstclasses.com, www.satpinnacle.com +91-9140474462, +91-8106429979 |8


Std. 10th |MATHEMATICS

HCF and LCM of two Positive Integers (Using Fundamental Theorem of Arithmetic)
We can find HCF and LCM of two positive integers using the prime factorisation method. Now, the
prime factorisation of a positive integer is nothing but its all about using the Fundamental Theorem
of Arithmetic.

Question 2:

Find the HCF and LCM of 35 and 115 by prime factorisation method.

Sol: 35  5  7
115  5  23
As studied in earlier classes
HCF (35, 115) = 5

LCM (35, 15) = 5  7  23 = 805.


Also, for two positive numbers a and b.
HCF (a, b)  LCM (a, b) = a  b.
We can use the above result to find one unknown quantity out of four if three of them are known.

Question 3:

Find HCF of 315 and 462 and hence find their LCM.

Sol: 315  32  7  5

462  2  3  7 11
HCF (315, 462) = 3  7 = 21.
Now, [product of two numbers = their HCF  their LCM}
HCF  LCM = 315  462
315  462
LCM = = 6930
21
 LCM (315, 462) = 6930.

Question 4:
Find the LCM and HCF of 6 and 20 by the prime factorisation method.?
Sol: We have : = 6  21  31 and 20 = 2  2  5  22  51

www.mstclasses.com, www.satpinnacle.com +91-9140474462, +91-8106429979 |9


Std. 10th |MATHEMATICS

You can find HCF(6, 20) = 2 and LCM(6, 20) = 2  2  3  5  60 , as done in your
earlier classes.
Note that HCF(6, 20) = 21 = Product of the smallest power of each common prime
factor in the numbers.
LCM (6, 20) = 22  31  51 = Product of the greatest power of each prime factor,
involved in the numbers.
From the example above, you might have noticed that HCF(6, 20)  LCM(6, 20) = 6 × 20. In fact,
we can verify that for any two positive integers a and b, HCF (a, b) × LCM (a, b) = a × b. We can
use this result to find the LCM of two positive integers, if we have already found the HCF of the two
positive integers.
Question 5:
Find the HCF of 96 and 404 by the prime factorisation method. Hence, find their LCM.
Sol: The prime factorisation of 96 and 404 gives:
96  25  3, 404  22 101
Therefore, the HCF of these two integers is 22  4 .
96  404 96  404
Also, LCM (96, 404) =  = 9696.
HCF  96, 404  4
Question 6:
Find the HCF and LCM of 6, 72 and 120, using the prime factorisation method.
Sol: We have:

6  2  3,72  23  32 ,120  23  3  5
Here, 21 and 31 are the smallest powers of the common factors 2 and 3, respectively.
So, HCF (6, 72, 120) = 21  31  2  3  6
23 ,32 and 51 are the greatest powers of the prime factors 2, 3 and 5 respectively involved in the three
numbers.
So, LCM (6, 72, 120) = 23  32  51  360
Remark: Notice, 6  72 120  HCF (6, 72, 120)  LCM (6, 72, 120). So, the product of three
numbers is not equal to the product of their HCF and LCM.

www.mstclasses.com, www.satpinnacle.com +91-9140474462, +91-8106429979 |10


Std. 10th |MATHEMATICS

Exercise 2:

1. Find HCF and LCM of the following pairs of numbers:


(a) 98 and 70; (b) 112 and 64
2. There are 3 alarms A, B, and C. A rings after every 3 hours, B rings after every 4 hours and C rings
after every 6 hours. Find the time interval after which the 3 alarms ring together after ring once
together.
3. Find the largest number that will divide 398, 436 and 542 leaving remainders 7, 11 and 15
respectively.
4. Express each number as a product of its prime factors:
(i) 140 (ii) 156
(iii) 3825 (iv) 5005
(v) 7429
5. Find the LCM and HCF of the following pairs of integers and verify that LCM  HCF = product of
the two numbers.
(i) 26 and 91
(ii) 510 and 92
(iii) 336 and 54
6. Find the LCM and HCF of the following integers by applying the prime factorisation method.
(i) 12, 15 and 21 (ii) 17, 23 and 29
(iii) 8, 9 and 25
7. Given that HCF (306, 657) = 9, find LCM (306, 657).
8. Check whether 6 n can end with the digit 0 for any natural number n.
9. Explain why 7  11  13 + 13 and 7  6  5  4  3  2  1 + 5 are composite numbers.
10. There is a circular path around a sports field. Sonia takes 18 minutes to drive one round of the field,
while Ravi takes 12 minutes for the same. Suppose they both start at the same point and at the same
time, and go in the same direction. After how many minutes will they meet again at the starting point?

www.mstclasses.com, www.satpinnacle.com +91-9140474462, +91-8106429979 |11


Std. 10th |MATHEMATICS

Revisiting Irrational Numbers


In Class IX, you were introduced to irrational numbers and many of their properties. You studied
about their existence and how the rational and the irrationals together made up the real numbers. You
even studied how to locate irrationals on the number line. However, we did not prove that they were
irrationals. In this section, we will prove that 2, 3, 5 and, in general, p is irrational, where p
is a prime. One of the theorems, we use in our proof, is the Fundamental Theorem of Arithmetic.
p
Recall, a number ‘s’ is called irrational if it cannot be written in the form , where p and q
q
are integers and q  0 . Some examples of irrational numbers, with which you are already familiar,
are:

2
2, 3, 15, ,  , 0.10110111011110…, etc.
3
Before we prove that 2 is irrational, we need the following theorem, whose proof is based on the
Fundamental Theorem of Arithmetic.
Theorem 1: Let p be a prime number. If p divides a2, then p divides a, where a is a positive integer.
Proof: Let the prime factorisation of a be as follows:
a  p1p 2 … p n , where p1 , p 2 , …, p n are primes, not necessarily distinct.

Therefore, a 2   p1p 2 ...p n   p1p 2 ,..., p n  = p12 p22 ...p2n .

Now, we are given that p divides a 2 . Therefore, from the Fundamental Theorem of
Arithmetic, it follows that p is one of the prime factors of a 2 . However, using the uniqueness part of
the Fundamental Theorem of Arithmetic, we realise that the only prime factors of a 2 are p1 , p2 ,..., pn .
So, p is one of p1 , p2 ,..., pn .

Now, since a  p1p2 ...pn , p divides a.

We are now ready to give a proof that 2 is irrational.


The proof is based on a technique called ‘proof by contradiction’. (This technique is discussed in
some detail in Appendix 1).
Theorem 2: 2 is irrational.

Proof: Let us assume, to the contrary, that 2 is rational.

r
So, we can find integers r and s   0  such that 2 .
s
www.mstclasses.com, www.satpinnacle.com +91-9140474462, +91-8106429979 |12
Std. 10th |MATHEMATICS

Suppose r and s have a common factor other than 1. Then, we divide by the common factor to
a
get 2  , where a and b are coprime.
b
So, b 2  a .
Squaring on both sides and rearranging, we get 2b2  a 2 . Therefore, 2 divides a 2 .
Now, by Theorem 1.3, it follows that 2 divides a.
So, we can write a = 2c for some integer c.
Substituting for a, we get 2b2  4c2 , that is, b2  2c2 .
This means that 2 divides b 2 and so 2 divides b.
Therefore, a and b have at least 2 as a common factor.
But this contradicts the fact that a and b have no common factors other than 1.
This contradiction has arisen because of our incorrect assumption that 2 is rational.
So, we conclude that 2 is irrational.
Question 1:

Prove that 3 is irrational.

Sol: Let us assume, to the contrary, that 3 is rational.


a
That is, we can find integers a and b   0  such that 3 .
b
Suppose a and b have a common factor other than 1, then we can divide by the common factor, and
assume that a and b are coprime.

So, b 3  a .
Squaring on both sides, and rearranging, we get 3b2  a 2 .
Therefore, a 2 is divisible by 3, and by Theorem 1. 3, it follows that a is also divisible by 3.
So, we can write a = 3c for some integer c.
Substituting for a, we get 3b2  9c2 , b2  3c2 .
This means that b 2 is divisible by 3, and so b is also divisible by 3.
Therefore, a and b have at least 3 as a common factor.
But this contradicts the fact that a and b are coprime.

This contradiction has arisen because of our incorrect assumption that 3 is rational.

So, we conclude that 3 is irrational.

www.mstclasses.com, www.satpinnacle.com +91-9140474462, +91-8106429979 |13


Std. 10th |MATHEMATICS

In Class IX. We mentioned that:


 the sum or difference of a rational and an irrational number is irrational and
 the product and quotient of a non-zero rational and irrational number is irrational.
We prove some particular cases here.
Question 2:

Show that 5  3 is irrational.

Sol: Let us assume, to the contrary, that 5  3 is rational.


a
That is we can find coprime a and b  b  0  such that 5  3  .
b
a
Therefore, 5   3
b
a 5b  a
Rearranging this equation, we get 3  5  .
b b
a
Since a and b are integers, we get 5  is rational, and so 3 is rational.
b

But this contradicts the fact that 3 is irrational.

This contradiction has arisen because of our incorrect assumption that 5  3 is rational.

So, we conclude that 5  3 is rational.


Question 3:

Show that 3 2 is irrational.


Sol: Let us assume, to the contrary, that 3 2 is rational.
a
That is, we can find coprime a and b  b  0  such that 3 2  .
b
a
Rearranging, we get 2 .
3b
a
Since 3, a and b are integers, is rational, and so 2 is rational.
3b
But this contradicts the fact that 2 is irrational.
So, we conclude that 3 2 is irrational.

www.mstclasses.com, www.satpinnacle.com +91-9140474462, +91-8106429979 |14


Std. 10th |MATHEMATICS

Exercise 3:

1. Prove that 5 is irrational.

2. Prove that 3  2 5 is irrational.

3. Prove that the following are irrationals:


1
(i) (ii) 7 5
2

(iii) 6 2

Revisiting Rational Numbers and Their Decimal Expansions


Any rational number can be represented either in terminating decimal or non-terminating recurring
decimals. In case of terminating decimal representation, the remainder becomes zero.
It can be seen that any rational number, whose decimal expansion terminates, can be expressed as a
rational number whose denominator is a multiple of 10. For e.g.
78 3027
0.78  0.3027  and so on.
102 104
Theorem 1: Let x be a rational number whose decimal expansion terminates. Then x can be expressed in
p n m
the form , where p and q are coprime, and the prime factorisation of q is of the form 2 5 ,
q
where n, m are non-negative integers.
You are probably wondering what happens the other way round in Theorem.
p
That is, if we have a rational number of the form , and the prime factorisation of q is of the form
q
p
2n 5m , where n, m are non-negative integers, then does have a terminating decimal expansion?
q
Let us see if there is some obvious reason why this is true. You will surely agree that any
a
rational number of the form , where b is a power of 10, will have a terminating decimal expansion.
b
p n m
So it seems to make sense to convert a rational number of the form , where q is of the form 2 5 ,
q
a
to an equivalent rational number of the form , where b is a power of 10. Let us go back to our
b
examples above and work backwards.

www.mstclasses.com, www.satpinnacle.com +91-9140474462, +91-8106429979 |15


Std. 10th |MATHEMATICS

3 3 3  53 375
(i)    = 0.375
8 23 23  53 103
13 13 13  23 104
(ii)    = 0.104
125 53 23  53 103
7 7 7  53 875
(iii)  4  4 4  4 = 0.0875
80 2  5 2  5 10
14588 22  7  521 26  7  521 233408
(iv)    = 23.3408
625 54 24  54 104
p
So, these examples show us how we can convert a rational number of the form , where q is of the
q
a
form 2n 5m , to an equivalent rational number of the form , where b is a power of 10. Therefore, the
b
decimal expansion of such a rational number terminates. Let us write down our result formally.
p
Theorem 2: Let x  be a rational number, such that the prime factorisation of q is of the form 2n5m ,
q
where n, m, are non-negative integers. Then x has a decimal expansion which terminates.
We are now ready to move on to the rational numbers whose decimal 0.1428571
expansions are non-terminating and recurring. Once again, let us look at 7 10
an ex ample to see what is going on. 7

We refer to example 5, Chapter 1, from your Class IX Textbook, namely, 30


1 28
. Here, remainders are 3, 2, 6, 4, 5, 1, 3, 2, 6, 4, 5, 1, … and divisor is
7 20
7. 14
Notice that the denominator here, i.e. 7 is clearly not of the form 2n5m. 60
1 56
Therefore, from Theorem 1 and 2, we know that will not have a
7 40
terminating decimal expansion.
35
Hence, 0 will not show up as a remainder (Why?), and the remainder 50
will start repeating after a certain stage. So, we will have a block of 49
1
digits, namely, 142857, repeating in the quotient of . 10
7
7
30

www.mstclasses.com, www.satpinnacle.com +91-9140474462, +91-8106429979 |16


Std. 10th |MATHEMATICS

1
What we have seen, in the case of , is true for any rational number not covered by Theorems
7
1.5 and 1.6. For such numbers we have:

p
Theorem 3: Let x  , where p and q are coprimes, be a rational number, such that the prime
q
n m
factorisation of q is not of the form 2 5 , where n, m are non-negative integers. Then, x has
a decimal expansion which is non-terminating repeating (recurring).

Question 1:

Which of the following rational numbers can be represented as terminating decimals?


(a) 3/5
(b) 2/13
(c) 7/20
Sol: We know that a rational number can be represented in the form of p/q, where p and q do not have any
common divisor. Then p/q can be represented as terminating decimal if q has only divisor 2 or 5 or
both.

3 p
(a)  q  5  51
5 q
 3/5 can be represented as terminating decimals.

2 p
(b)  here q = 13.
13 q
2
Since divisor of q is only 13, hence can be represented as terminating decimal.
13

7 p
(c) 
20 q

q  20  4  5  22  5

Hence, divisor of q can be represented in terms of 2 and 5.


7
Hence, can be represented as terminating decimal.
20

www.mstclasses.com, www.satpinnacle.com +91-9140474462, +91-8106429979 |17


Std. 10th |MATHEMATICS

Exercise 4:

1. Without actually performing the long division, state whether the following rational numbers will
have a terminating decimal expansion or a non-terminating repeating decimal expansion:
13 17 64
(i) (ii) (iii)
3125 8 455
15 29 23
(iv) (v) (vi)
1600 343 2352
129 6 35
(vii) (viii) (ix)
2257 75 15 50
77
(x)
210
2. Write down the decimal expansions of those rational numbers in Question 1 above which have
terminating decimal expansions.
3. The following real numbers have decimal expansions as given below. In each case, decide whether
p
they are rational or not. If they are rational, and of the form , what can you say about the prime
q
factors of q?
(i) 43.123456789 (ii) 0.120120012000120000. . .
(iii) 43.123456789

Unit Digit of Any Power


1. The unit digit of any power of 0, 1, 5 and 6 is 0, 1, 5 and 6 respectively.
For example:

The unit digit of  20  is 0.


53
(i)

The unit digit of  81


43
(ii) is 1

The unit of digit of  65 


123
(iii) is 5

The unit digit of  216 


273
(iv) is 6.

2. The unit digit of (4) odd number is 4.


The unit digit of (4) even number is 6.

www.mstclasses.com, www.satpinnacle.com +91-9140474462, +91-8106429979 |18


Std. 10th |MATHEMATICS

The unit digit of (9) odd number is 9.


The unit digit of (9) even number is 1.
For example:

The unit digit of  34  is 4.


57

The unit digit of  29  is 1.


32

3. The unit digit of the power of 2, 3,7 and 8 follow a cyclic pattern i.e. they repeat after every
4 steps therefore to find the unit digit of any power of 2, 3, 7 and 8 we first divide the power
by 4 and look at the remainder, the remainder can be 0, 1, 2 or 3 and to find the unit digit use
the following table.

Remainder Unit digit of 2 Unit digit of 3 Unit digit of 7 Unit digit of 8


1 2(21  2) 3(31  3) 7(71  7) 8(81  8)
2 4(22  4) 9(32  9) 9(7 2  49) 4(82  64)
3 8(23  8) 7(33  27) 3(73  343) 2(83  512)
0 6(24  16) 1(34  81) 1(7 4  2401) 6(84  4096)

For example:
31
The unit digit of (72) is ……..

Step 1: 431  4 , remainder is 3.

Step 2: 23  8  unit digit of (72)431 is 8.


1024
The unit digit of (128) is

Step 1: 1024  4 , remainder is 0.

Step 2: 84  4096  unit digit of (128)1024 is 6.


Question 1:

Find the unit digit of 14    29 


124 123

Sol: Unit digit of 14 


124
  29 
123

= unit digit of 4124  9123 

www.mstclasses.com, www.satpinnacle.com +91-9140474462, +91-8106429979 |19


Std. 10th |MATHEMATICS

The unit digit of  4 


124
is 6.

The unit digit of  9 


123
is 9.

Now 6  9 = 54

 Unit digit of 14    29  is 4.


124 123

Finding the Remainder in Divisions Involving Power of Numbers.


The method to find remainder in divisions involving power of numbers is explained below with the
help an example.

Question 2:

Find the remainder of 343 when divided by 4.


Sol: Firstly, we find the pattern that the remainder follow when the successive powers of 3 are divided by
4.

Remainder of 31 when divided by 4 = 3.

Remainder of 32 when divided by 4 = 1.

Remainder of 33 when divided by 4 = 3.


 The remainder repeats after 2 steps and it is 3 when exponent of 3 is odd and it is 1 when exponent
of 3 is even.  Requited remainder = 3.
Rules Pertaining to an + bn or an – bn:

Rule 1: a n  bn is always divisible by a  b , n is even or odd.

Rule 2: a n  bn is divisible by a + b if n is even.

Rule 3: a n  bn is divisible by a + b if n is odd.

Question 3:

Find the remainder when  23  15 


17 17
is divided by 19.

By Rule 3:  23  15  is divisible 23 + 15 = 38 is also divisible by 19.


17 17
Sol:

 when  23  15 is divisible by 19 the remainder will be zero.


17 17

www.mstclasses.com, www.satpinnacle.com +91-9140474462, +91-8106429979 |20


Std. 10th |MATHEMATICS

www.mstclasses.com, www.satpinnacle.com +91-9140474462, +91-8106429979 |21


Std. 10th |MATHEMATICS

KEY TO EXERCISES

Exercise 1:
2. 4
5. (i) 45 (ii) 196 (iii) 51
6. An integer can be of the form 6q, 6q + 1, 6q + 2, 6q + 3, 6q + 4 or 6q + 5.
7. 8 columns
8. An integer can be of the form 3q, 3q + 1 or 3q + 2. Square all of these integers.
9. An integer can be of the form 9q, 9q + 1, 9q + 2, 9q + 3,…, or 9q + 8.
Exercise 2:
1. (a) 14,490 (b) 16,448
2. 12 hours 3. 17
4. (i) 22  5  7 (ii) 22  3 13 (iii) 32  52 17
(iv) 5  7 1113 (v) 17 19  23
5. (i) LCM = 182; HCF = 13 (ii) LCM = 23460; HCF = 2
(iii) LCM = 3024; HCF = 6
6. (i) LCM = 420; HCF = 3 (ii) LCM = 11339; HCF = 1
(iii) LCM = 1800; HCF = 1
7. 22338 10. 36 minutes
Exercise 4:
1. (i) Terminating (ii) Terminating
(iii) Non-terminating repeating (iv) Terminating
(v) Non-terminating repeating (vi) Terminating
(vii) Non-terminating repeating (viii) Terminating
(ix) Terminating (x) Non-terminating repeating
2. (i) 0.00416 (ii) 2.125
(iv) 0.009375 (vi) 0.115
(viii) 0.4 (ix) 0.7
3. (i) Rational, prime factors of q will be either 2 or 5 or both only.
(ii) Not rational
(iii) Rational, prime factors of q will also have a factor other than 2 or 5.
www.mstclasses.com, www.satpinnacle.com +91-9140474462, +91-8106429979 |22
Std. 10th |MATHEMATICS

SOLVED PROBLEMS
SUBJECTIVE TYPE
Section - A
Problem 1: Find HCF of 3675 and 42 using Euclid’s division algorithm.
Solution: Step 1: 3675 = 42  87 + 21
Remainder is not zero. Now 42 becomes the new dividend and 21 becomes the new divisor.
Step 2: 42 = 21  2+0
Remainder becomes zero here.
 the divisor at this step is the HCF of 3675 and 42
 HCF (3675, 42) = 21.
Problem 2: Using Euclid’s division lemma, prove that difference of squares of 2 odd natural numbers is
a multiple of 8.
Solution: Let the 2 odd natural numbers be m and n.
By division lemma.
m  2u  1, n  2v  1

m 2  n 2   m  n  m  n 

= [2u + 1 – (2v + 1)] [2u + 1 + 2v + 1]


= 4(u – v) (u + v + 1) …(i)
Now, if u and v are both even or both odd, then u – v will also be even and hence (i) is a
multiple of 8.
Otherwise, when one of u and v is even and one is odd, then u + v + 1 will be even and hence
(i) is a multiple of 8.
Problem 3: Check whether 7 n can end with the digit ‘O’ for any natural number n.
Solution: If 7 n ends with zero, then 2 or 5 must be a factor of 7 n . But by fundamental theorem of
arithmetic, 7 n has only factor 7.
 7 n cannot end with a zero for any natural number n.
Problem 4: Find LCM and HCF of the following numbers using prime factorisation:
(a) 14, 22, 32 (b) 18, 81, 90
Solution: (a) 14 = 2 7
22 = 2  11

www.mstclasses.com, www.satpinnacle.com +91-9140474462, +91-8106429979 |23


Std. 10th |MATHEMATICS

32 = 25
HCF = 2, LCM = 32 × 11 × 7 = 2464
(b) 18, 81, 90
18 = 2  32
81 = 34
90 = 2  32  5
HCF = 32  9
LCM = 34  2  5 = 810.
Problem 5: Given that HCF (189, 144) = 9, find their LCM.
Solution: HCF  LCM = product of 2 numbers
9  LCM = 189  144
189 144
 LCM = = 3024.
9
Problem 6: If a is an irrational number, then prove that –a is also an irrational number.
Solution: Let a be an irrational number. Say, if –a is not an irrational number then –a is a rational
number. Now, –(–a) = a is an irrational number. But we know that if (–a) is a rational number
then –(–a) = a will also be a rational number, which is a contradiction. Hence, our supposition
is wrong. Therefore –a is an irrational number.

Problem 7: Prove that 2  2 is not a rational number.

Solution: Let a  2  2 be a rational number. Squaring both sides,

a2  4  2  2  2  2  6  4 2

a2  6
 = 2
4
a is a rational number  a 2 is also a rational number.

a2  6
 is also a rational number.
4
But we know that an irrational number can never be equal to a rational number.

a2  6
Hence, 2 is not possible.
4

www.mstclasses.com, www.satpinnacle.com +91-9140474462, +91-8106429979 |24


Std. 10th |MATHEMATICS

Because 2 is an irrational number, our supposition 2  2 is a rational number is wrong.


Hence, 2  2 is an irrational number.
Problem 8: Without performing the long division process state whether the following rational numbers
will have terminating decimal expansion or a non-terminating repeating decimal expansion.
17 64
(i) (ii)
8 455
29 15
(iii) (iv)
343 1600
13 23
(v) (vi)
3125 2352
17 17
Solution: (i) 
8 23
So, denominator 8 is of form 2m  5n , where m, n are non-negative integers.
17
Hence, has a terminating decimal expansion.
8
64 64
(ii)  Non-terminating repeating decimal expansion
455 5  7 13
29 29
(iii)  Non-terminating repeating decimal expansion
343 35
15 3 3 3
(iv)    6 1 Terminating decimal expansion
1600 320 64  5 2  5
13 13
(v)  5 Terminating decimal expansion
3125 5
23
(vi) Terminating decimal expansion
2352

Problem 9: Let a, b, c and d be positive rational such that a  b  c  d , then show that either a  c
and b = d.
or b and d are squares of rationals.
Solution: Case I: a = c

As a  b  c  d

 b d
 b=d
Case II: a  c

www.mstclasses.com, www.satpinnacle.com +91-9140474462, +91-8106429979 |25


Std. 10th |MATHEMATICS

then, there exists a rational number


x such that a  c  x

Now a  b  c  d

 cx b  c d

 x b  d …(i)

 x  b  2 bx  d
2
(on squaring both sides)

d  b  x2
 b = rational number
2x
Hence, b is square of a rational number again from (i)

d  bx
b is square of a rational number
Hence, d is also square of a rational number.

Problem 10: Prove that 5  3 is an irrational number.

Solution: Let us suppose that 5  3 is a rational number

Then 5  3 = a , where b  0 and a, b are integers


b
2

  a 
2
 5    3
b 

a 2  3b 2  2ab 3
 5=
b2

a 2  3b2  5b2
  3
2ab
a 2  2b 2
  3
2ab
a 2  2b 2
Since, a and b are integers hence, must be a rational number which is a
2ab
contradiction.

www.mstclasses.com, www.satpinnacle.com +91-9140474462, +91-8106429979 |26


Std. 10th |MATHEMATICS

Hence, 5  3 is irrational.
Problem 11: Which is greater?
3 4
(i) 2 or 3 (ii) 3 or 4 10 (iii) 5 or 3
4

Solution: (i) LCM of 2 and 3 is 6.

Thus, 2  6 23  6 8 and 3
3  6 32  6 9

 69 2
Hence, 3
3 2
(ii) LCM of 1 and 4 is 4

Thus, 3  4 32  4 9 and 4
10  4 10

 4
10  3
(iii) LCM of 4 and 3 is 12

Thus, 4
5  12 53  12 125 and 4
3  12 44  12 256

Hence, 3
445
Problem 12: Simplify and express the result in its convert form 3
4  3 22 .

Solution: 3
4  3 22 =
3
4  22  3 23 11  2 3 11

Problem 13: 3 
5  5 2 4 5  3 2 simplify it. 
Solution: 3 
5 5 2 4 5 3 2 
= 12   5  5  
2 2
 9 5  2  20 2  5  15



= 12  5  9 10  20 10  15  2 
=  60  30    9  20  10  30  11 10

p
Problem 14: Represent 0.57 in the form of .
q

www.mstclasses.com, www.satpinnacle.com +91-9140474462, +91-8106429979 |27


Std. 10th |MATHEMATICS

p
Solution: = 0.57 …(i)
q
p
100  57.57 …(ii)
q
Subtract (i) by (ii)
p
99  57
q
p 57
 .
q 99
3
Problem 15: Which is greater 3 or 5?
Solution: LCM of 2 and 3 is 6

Thus, 3  6 33  6 27
3
5  6 52  6 25

 335

Section - B
1
If y  3  3 , then value of y  9y  27y .
3 3 2
Problem 1:
1
Solution: Given y  33  3
1
 y  3  33
Taking the cubes on both sides
 y  9y  27y  27  3
3 2

 y  9y  27y  30
3 2

2 x2 3 x3 6 x4 10
Problem 2: Simplify this 2x 3x 6x 9x

x x3 x6 x10
 3
2 3  6x  9x
6 10
x x
Solution:

2  3 6  9
= 18

www.mstclasses.com, www.satpinnacle.com +91-9140474462, +91-8106429979 |28


Std. 10th |MATHEMATICS

Problem 3: 7  2 6  7  2 6 , simplify to p  q form

 
2
Solution: 7  2 6  6 1 2 6  6 1

 
2
7  2 6  6 1 2 6  6 1

By adding both

= 6  1  6 1  2 6
1 2
Problem 4: If x  1  53  5 3 , then find the value of x 3  3x 2  12x  16 .
1 2
Solution: x  1  53  5 3
1 2
x  1  53  5 3
Cubing both sides we get

 x  1  5  52  3  5  x  1
3

x 3  1  3x  x  1  30  15x  15

x 3  3x 2  12x  16  0
p2  1
Problem 5: If p  7  4 3 , then value of
7p
Solution: If p  7  4 3
1 1

p 74 3
By rationalizing
1 74 3

p 49  48
1
 74 3
p
1 1
 p  74 374 3 p   14
p p
p2  1
Now,  14
p

www.mstclasses.com, www.satpinnacle.com +91-9140474462, +91-8106429979 |29


Std. 10th |MATHEMATICS

p2  1
  2.
7p
OBJECTIVE
Level - I
Multiple Choice Questions (Single Option Correct)

Problem 1: Given that HCF (306, 657) = 9, find LCM (306, 657)
(a) 22333 (b) 22338
(c) 33228 (d) none of these
Solution: (B). We know HCF (a, b) × LCM (a, b) = a × b
i.e. 9 × LCM (306, 657) = 306 × 657
306  657
i.e. LCM (306, 657) = = 22338.
9
Problem 2: Find the number of zeroe’s in the end of 6 n for any natural number n.
(a) 10 (b) 100
(c) 0 (d) none of these
Solution: (C). 6 n ends with 0, then 2 and 5 must be a factor of 6 n . But by fundamental theorem of
arithmetic, 6 n has factors 2 and 3.
 6 n cannot end with 0, for any n  N .
Problem 3: By applying division lemma, find out HCF (36168, 210).
(a) 8 (b) 9
(c) 6 (d) none of above
Solution: (C). 36168 = 210 × 172 + 48
210 = 48 × 4 + 18
48 = 18 × 2 + 12
18 = 12 × 1 + 6
12 = 6 × 2 + 0 remainder, 0 came at the last step.
 HCF (36168, 210) = 6.
Problem 4: As express each of the following numbers as a product of its prime factors find minimum
value of ‘n’ in 7 n

www.mstclasses.com, www.satpinnacle.com +91-9140474462, +91-8106429979 |30


Std. 10th |MATHEMATICS

60025, 756, 6615


(a) 1 (b) 2
(c) 4 (d) none of these

Solution: (A). 60025 = 52  7 4

756 = 22  33  7

6615 = 33  5  7 2
Problem 5: As express each of the following numbers as a product of its prime factors find HCF of the
following:
60025, 756, 6615
(a) 35 (b) 120
(c) 7 (d) none of these

Solution: (C). 60025 = 52  7 4

756 = 22  33  7

6615 = 33  5  7 2
Problem 6: As express each of the following numbers as a product of its prime factors find the maximum
value of k in 3k .
60025, 756, 6615
(a) 2 (b) 3
(c) 1 (d) none of these

Solution: (B). 60025 = 52  7 4

756 = 22  33  7

6615 = 33  5  7 2

Problem 7: Simplify: 6 3  5 12 .

(a) 3 16 (b) 16 3

(c) 15 12 (d) none of above

Solution: (B). 6 3  5 12

www.mstclasses.com, www.satpinnacle.com +91-9140474462, +91-8106429979 |31


Std. 10th |MATHEMATICS

= 6 3  5 43

= 6 3  10 3

= 16 3

(Fill in the Blanks)

Problem 8: HCF of 81 and 237 is _____________.


Solution: 237 = 81 × 2 + 75
81 = 75 × 1 + 6
75 = 6 × 12 + 3
6=3×2+0
Hence, HCF = 3.
Problem 9: If HCF of 210 and 55 is 5, then LCM of 210 and 55 = ____________.
Solution: HCF × LCM = 210 × 55
 5 × LCM = 210 × 55
 LCM = 210 × 11 = 2310.
(True or False)

Problem 10: Since HCF of 2 and 4 is 2. Hence, HCF of 4 and 16 is 4.

Solution: (True) If HCF of a and b is c, then HCF of a 2 and b 2 is c2 .


Problem 11: If HCF of a and b is h and LCM of a and b is l, then h : a :: b : l.
Solution: (True) HCF × LCM = a × b.
21
Problem 12: has a terminating decimal expansion.
175
21 21 3
Solution: (True)  2  2
175 5  7 5

Level – II

Problem 1: HCF of 87 and 99 is

www.mstclasses.com, www.satpinnacle.com +91-9140474462, +91-8106429979 |32


Std. 10th |MATHEMATICS

(a) 9 (b) 3
(c) 27 (d) none of these
Solution: (B). 87 = 3 × 29

99 = 32 11
 HCF = 3.
Problem 2: A number of the form 6q  2  q  N  is

(a) always odd (b) always even


(c) depends on q (d) none of these
Solution: (B). 6q + 2 = 2(3q + 1)
i.e. 6q + 2 has 2 as a factor.
 always even.
Problem 3: LCM of 93 and 102 is
(a) 3162 (b) 9486
(c) 1581 (d) none of these
Solution: (A). 93 = 3 × 31
102 = 3 × 34
 LCM = 3 × 31 × 34
= 3162.
Problem 4: Which of the following rational numbers has a terminating decimal expansion?
17 41
(a) (b)
343 63

81
(c) (d) none of these
95

Solution: (D). If the denominator has a prime factorisation of the form of 2m5n , then the rational number
has a terminating decimal expansion.

Here, 343  73

63 = 32  7

www.mstclasses.com, www.satpinnacle.com +91-9140474462, +91-8106429979 |33


Std. 10th |MATHEMATICS

95 = 5 19

None of the denominator has factorisation of the form 2m5n .


Problem 5: Which of the following real numbers are not rational?

(a) 0.30100200040000 … (b) 0.123

(c) 0.1231437 (d) none of these


Solution: (A). Real numbers whose decimal representation is non-terminating and non-repeating are
irrational numbers.

1
Problem 6: If x  , then x 2  10x  1 =
5 2 6
(a) 1 (b) –1
(c) 0 (d) 10

1
Solution: (C). x 
5 2 6

52 6
x (by rationalizing)
25  24

 x = 52 6

1
x  10
x

x 2  10x  1  0

www.mstclasses.com, www.satpinnacle.com +91-9140474462, +91-8106429979 |34


Std. 10th |MATHEMATICS

ADD TO YOUR KNOWLEDGE


(i)

When  ak  1 is divided by a then


n

(a) remainder is 1 if n is even.


(b) remainder is (a – 1) if n is odd

 a n  bn is always divisible by a + b if n is odd.

 a n  bn is always divisible by a + b if n is even.

 a n  bn is always divisible by a – b if n is either even or odd.

(ii)
LCM of numerators
 LCM of fractions =
HCFof denominators

HCFof numerators
 HCF of fractions =
LCM of denominators

(iii) Product of two numbers = Product of their LCM and HCF.


(iv)

 Greatest number that will divide x, y and z leaving remainder p, q and r respectively.
Required number = HCF of (x – p), (y – q), (z – r)

 Greatest number that will divide x, y and z leaving the same remainder in each case.

Required number = HCF of x  y , y  z and z  x

 Least number which when divided by x, y and z leaves the remainder p, q and r respectively
and x  p  y  q  z  r  k

Required number = (LCM of x, y and z) – k.

 Every even positive integer can be written as 2n and every odd integer can be written as
2n  1 where n is any whole number.

 Any positive integer can be written as 3n or 3n + 1 or 3n + 2 for some integer n.


Square of any integer can be written as 3m or 3m + 1.

www.mstclasses.com, www.satpinnacle.com +91-9140474462, +91-8106429979 |35


Std. 10th |MATHEMATICS

CHAPTER PRACTICE PROBLEMS

1. If the HCF 408 and 1032 is expressible in the form 1032 × m – 408 × 5. Find m.

2. Find the largest number which exactly divides 280 and 1245 and leaves remainders of 4 and 3.

3. Prove that there is no natural number for which 4 n ends with the digit zero.

4. There are two positive integers X and Y. When X is divided by 237, the remainder is 192. When Y
is divided by 117 the quotient is the same but the remainder is 108. Find the remainder when the sum
of X and Y is divided by 118.

5. Find the remainder when 4333  2333 is divided by 5.

Prove that if x and y are odd positive integers, then x  y is even but not divisible by 4.
2 2
6.

7. Prove that 2  3 is irrational.

8. A number when divided by a divisor leaves a remainder of 27. Twice the number divided by the same
divisor leaves a remainder of 3. Find the divisor?

9. If N is two-digit prime number. When digits are interchanged, we get another prime number M. If
N  M  176 then find N – M.

10. A number when divided by a divisor leaves a remainder of 5 and when divided by twice the divisor
leaves a remainder of 45. Find the divisor.

www.mstclasses.com, www.satpinnacle.com +91-9140474462, +91-8106429979 |36


Std. 10th |MATHEMATICS

ASSIGNMENT

Section - A
SUBJECTIVE

1. State Euclid’s division lemma.


2. Find LCM of 36, 40, 48.
3. State the fundamental theorem of arithmetic.
4. Write 98 as a product of its prime factors.
5. Write the prime factor of 2700.

p
6. Write the condition to be satisfied by q so that a rational number has a terminating decimal
q
expansion.
7. Which two of the following rational numbers have a terminating decimal representation?
81 91
(i) (ii)
80 81
64 97
(iii) (iv)
125 90

13
8. Without performing the long division process write whether the rational number has a
3125
terminating decimal or a non-terminating repeating decimal.
64
9. Write condition satisfied by 455 so that a rational number has a non-terminating repeating
455
decimal expansion.
10. Classify the following numbers as rational or irrational

(a) 3 6 (b) 

 
2
(c) 5 3 (d) 2.3030030003……

6  6 6  6 
7
(e) (f)
2 5

www.mstclasses.com, www.satpinnacle.com +91-9140474462, +91-8106429979 |37


Std. 10th |MATHEMATICS

13
11. Write whether the number has a terminating decimal or a non-terminating decimal.
625

325
12. Write whether the number has a terminating decimal or a non-terminating decimal.
625 52

13. Is 3  7 irrational?

14. Can a number of the form 5n have a zero at its end?

15. Check whether 6 n can end with the digit 0 for any natural number n.
16. Find HCF = 154 and 660.
17. Given HCF (273, 133) = 7, find their LCM.
18. Given that HCF (306, 657) = 9, find LCM (306, 657)
19. Find the HCF and LCM of 24, 36, 48 and hence check that HCF × LCM is not equal to product of
three given numbers.
20. Prove that HCF (24, 4) = HCF (12576, 4052).
21. Find the largest number which divides 62, 132 and 237 to leave the same remainder in each case.

22. Find the unit digit of 192013  112013  62013

23. What can you say about the prime factorization of the denominator of the rational number 34.5678 .
24. Show that every positive even integer is of the form 2n and every positive odd integer is of the form
2n + 1.
25. Find the least number which when divided by 2, 3, 4, 5, 6, leaves remainder of 1 in each case but
when divided by 7 leaves no remainder.
26. Show that any positive integer is of the from 3q or 3q + 1 or 3q + 2 for some integer q.
27. A garden has 48 guava trees, 60 pineapple trees and 96 mango trees. These have to be arranged in
show such that each row has same numbers of trees and all are of same type. Find the minimum
number of such rows that can be formed?

28. Show that 3  2 5 is irrational.

29. If the HCF of 210 and 55 is expressible in form 210 × 5 + 55y, find y.

www.mstclasses.com, www.satpinnacle.com +91-9140474462, +91-8106429979 |38


Std. 10th |MATHEMATICS

Section - B

1. Which among the following is greatest 7  3, 5  5, 6  2 ?


2. Express each of the following as a product of its prime factors:
(a) 6615 (b) 756 (c) 60025
3. Does there exist any irrational number in between any two rational numbers?
4. Find the least multiple of 7, which leaves a remainder of 4, when divided by 6, 9, 15 and 18.

   7  4 3 
1/2 1/2
5. Solve: 28  10 3

6. A man has 1044 candles. After burning, he can make a new candle from 9 stubs left behind. Find the
maximum number of candles that can be made.
7. The difference of 1025  7 and 1024  x is divisible by 3, find least positive x.
8. Find the least number that is divisible by all the natural numbers between 1 and 10. (Both inclusive)
9. Use Euclid’s division algorithm to find HCF of 96 and 294.
10. Which two of the following real numbers are irrational?
(a) 0.3421234676 … (b) 0.1010010001…
(c) 0.1372 (d) 0.24579
11. Show that any positive odd integer is of the form of 4q + 1 or 4q + 3 where q is any integer.
12. Two runners A and B halt after 3 hours and 5 hours respectively while running. After how many
hours will both of them halt together for the first time?
13. Prove that one and only one out of n, n + 2 and n + 4 is divisible by 3, where n is any positive integer.
14. Prove that one of any three consecutive positive integers must be divisible by 3.

15. For any positive integer n, prove that n 3  n is divisible by 6.


16. Show that one and only one out of n, n + 4, n + 8, n + 12 and n + 16 is divisible by 5, where n is any
positive integer.
17. Find the remainder when the product of any three consecutive integers is divided by 6.
18. Given two numbers 32 and 128. Find their HCF and LCM show that HCF × LCM = 32 × 128.
19. Find LCM of 6, 72 and 120 using the prime factorization method.
20. Explain why 7 1113  13 and 7  6  5  4  3  2 1  5 are composite numbers.

www.mstclasses.com, www.satpinnacle.com +91-9140474462, +91-8106429979 |39


Std. 10th |MATHEMATICS

21. What is the volume of sphere of radius 3 and hence conclude that the volume of this sphere is
terminating decimal or non-terminating decimal?
22. Show that any positive odd integer of the form 8p + 1 or 8p + 3 or 8p + 5 or 8p + 7 where p is some
integer.
9 12 18 21
23. Find the HCF of , , and .
10 25 35 40

24. Prove that 5 is an irrational number.

25. Prove that 5  7 is an irrational number.


26. Find the largest number which divides 245 and 1029 leaving remainder 5 in each case.
27. Find the largest number that divides 2053 and 967 and leaving a remainder 5 and 7 respectively.
28. Find the greatest numbers that will divide 455, 572 and 699 leaving remainders 4, 5 and 6
respectively.
29. 4 bells toll together at 9:00 a.m. They toll after 7, 8, 11 and 12 seconds respectively. How many times
will they toll together again in the next 3 hours?
30. Two numbers are in the ratio 17 : 13. If their HCF is 15. What are the numbers?
31. Three persons running around a circular track, can complete one revolution in 2, 4 and 5.5 hours
respectively. When will they meet at the starting point?

32. Show that any prime number can be expressed in the form of 6k  1 k  N  for number greater than
3.
33. How many numbers up to 100 are co-prime to 19?
34. Write whether every positive integer can be of the form 4q + 2, where q is an integer. Justify your
answer.
35. “The product of two consecutive positive integers is divisible by 2”. Is this statement true or false?
Give reasons.
36. “The product of three consecutive positive integers is divisible of 6”. Is this statement true or false”?
Justify your answer.
37. Write whether the square of any positive integer can be of the form 3m + 2, where m is a natural
number. Justify your answer.
38. A positive integer is of the form 3q + 1, q being a natural number. Can you write its square in any
form other than 3m + 1, i.e. 3m or 3m + 2 for some integer m? Justify your answer.
www.mstclasses.com, www.satpinnacle.com +91-9140474462, +91-8106429979 |40
Std. 10th |MATHEMATICS

39. The numbers 525 and 3000 are both divisible only by 3, 5, 15, 25 and 75. What is HCF (525, 3000)?
Justify your answer.

40. Explain why 3  5  7  7 is a composite number.


41. Can two numbers have 18 as their HCF and 380 as their LCM? Give reasons.
987
42. Without actually performing the long division, find if will have terminating or non-
10500
terminating (repeating) decimal expansion. Give reasons for your answer.
43. A rational number in its decimal expansion is 327.7081. What can you say about the prime factors of
p
q, when this number is expressed in the form ? Give reasons.
q
44. Show that the square of any positive integer is either of the form 4q or 4q + 1 for some integer q.
45. Show that cube of any positive integer is of the form 4m, 4m + 1 or 4m + 3, for some integer m.
46. Show that the square of any positive integer cannot be of the form 5q + 2 or 5q + 3 for any integer q.
47. Show that the square of any positive integer cannot be of the form 6m + 2 or 6m + 5 for any integer
m.
48. Show that the square of any odd integer is of the form 4q + 1, for some integer 1.

49. If n is an odd integer, then show that n 2  1 is divisible by 8.

Prove that if x and y are both odd positive integers, then x  y is even but not divisible by 4.
2 2
50.

51. Use Euclid’s division algorithm to find the HCF of 441, 567, 693.
52. Using Euclid’s division algorithm, find the largest number that divides 1251, 9377 and 15628 leaving
remainder 1, 2 and 3, respectively.

53. Prove that 3  5 is irrational.

54. Show that 12 n cannot end with the digit 0 or 5 for any natural number n.
55. On a morning walk, three persons step off together and their steps measure 40 cm, 42 cm and 45 cm,
respectively. What is the minimum distance each should walk so that each can cover the same distance
in complete steps?
257
56. Write the denominator of the rational number in the form 2m  5n , where m, n are non-negative
5000
integers. Hence, write its decimal expansion, without actual division.

www.mstclasses.com, www.satpinnacle.com +91-9140474462, +91-8106429979 |41


Std. 10th |MATHEMATICS

57. Prove that p  q is irrational, where p, q are primes.

58. Show that the cube of a positive integer of the form 6q + r, q is an integer and r = 0, 1, 2, 3, 4, 5 is
also of the form 6m + r.
Level - I
OBJECTIVE
Multiple Choice Questions (Single Option Correct)
1. Which of the following statements is false:
(a) sum of 2 irrational numbers is always irrational
(b) product of 2 irrational numbers may be rational or irrational
(c) sum of 2 irrational numbers may be rational or irrational
(d) none of these
2. Which of the following pairs of numbers are coprime?
(a) 13, 17 (b) 81, 18
(c) 108, 26 (d) 42, 56
3. HCF of 135 and 1575 is
(a) 45 (b) 5
(C) 3 (d) 1
4. Which of the following is irrational?

(a) 81 (b) 0.273

(c) 0.321010397 … (d) 0.3251


5. Which of the following statements is false?
(a) product of HCF and LCM of 3 numbers is equal to product of those 3 numbers

(b) a number of the form 5n  5 always ends with zero (n is a natural number)
(c) every real number is rational or irrational
93
(d) has terminating decimal representation
25

www.mstclasses.com, www.satpinnacle.com +91-9140474462, +91-8106429979 |42


Std. 10th |MATHEMATICS

6. 4.12 is equivalent to

103 138
(a) (b)
25 47
136
(c) (d) none of these
33
7. If p and q are two co-prime number, then their HCF is equal to
(a) p (b) q
(c) 1 (d) pq
8. The greatest number that will exactly divide 162, 257 and 324 leaving the remainder 6, 10 and 12
respectively, is
(a) 13 (b) 12
(c) 17 (d) 11

9. A rational number between 2 and 3 is:

2 3 2 3
(a) (b)
2 2
(c) 1.5 (d) 1.4

10. x n
 a n  is completely divisible by (x – a) when

(a) n is any natural number (b) n is prime


(c) n is an even number (d) n is an odd number
11. The LCM and HCF of two rational number are equal, then the numbers must be
(a) prime (b) co-prime
(c) composite (d) equal
12. The HCF and LCM of two numbers are 9 and 90 respectively. If one of the number is 45, then the
other number is

(a) 27 (b) 18
(c) 30 (d) none of these

www.mstclasses.com, www.satpinnacle.com +91-9140474462, +91-8106429979 |43


Std. 10th |MATHEMATICS

13. Which of the following rational number have terminating decimal?


16 5
(a) (b)
225 18
2 7
(c) (d)
21 250
14. If n is a natural number, then 6n  5n always ends with
(a) 1 (b) 3
(c) 5 (d) 7

If two positive integers a and b are expressible in the form a  pq and b  p q; p,q being prime
2 3
15.
number then LCM (a, b) is

(a) pq (b) p3q 3

(c) p3q 2 (d) p2q 2


16. For some integer m, every even integer is of the form
(a) m (b) m+1
(c) 2m (d) 2m + 1
17. For some integer q, every odd integer is of the form
(a) q (b) q+1
(c) 2q (d) 2q + 1
18. n 2  1 is divisible by 8, if n is
(a) an integer (b) a natural number
(c) an odd integer (d) an even integer
19. If the HCF of 65 and 117 is expressible in the form 65m – 117, then the value of m is
(a) 4 (b) 2
(c) 1 (d) 3
20. The largest number which divides 70 and 125, leaving remainders 5 and 8, respectively, is
(a) 13 (b) 65
(c) 875 (d) 1750

www.mstclasses.com, www.satpinnacle.com +91-9140474462, +91-8106429979 |44


Std. 10th |MATHEMATICS

If two positive integers a and b are written as a  x y and b  x y ; x, y are prime numbers, then
3 2 2 3
21.
HCF (a, b) is
(a) xy (b) x 2 y2
(c) x 3 y3 (d) xy 2
If two positive integers p and q can be expressed as p  ab and q  a b;a, b being prime numbers,
2 3
22.
then LCM (p, q) is
(a) ab (b) a 2 b2

(c) a 3b2 (d) a 3 b3


23. The product of a non-zero rational and an irrational number is
(a) always irrational (b) always rational
(c) rational or irrational (d) one
24. The least number that is divisible by all the numbers from 1 to 10 (both inclusive) is
(a) 10 (b) 100
(c) 504 (d) 2520
14587
25. The decimal expansion of the rational number will terminate after:
1250
(a) one decimal place (b) two decimal places
(c) three decimal places (d) four decimal places
(Match the following)
1. Match the following:

Column I Column II
(A) HCF of 92 and 21 is (i) 1008
(B) LCM of 1008 and 63 is (ii) 7224
(C) LCM of 1032 and 301 (iii) 1
(D) HCF of 203 and 145 (iv) 29

www.mstclasses.com, www.satpinnacle.com +91-9140474462, +91-8106429979 |45


Std. 10th |MATHEMATICS

2. Match the following:

Column I Column II
(A) Decimal representation of an irrational (i) Irrational number
number is always

 1 
2
(ii) Rational Number
(B)  2  
 3
(C) The decimal representation of 8/27 is (iii) Non-repeating, non-terminating
(D) Zero is (iv) Non-terminating

Level – II

Multiple Choice Questions (One or More than One option correct)

1 1 3
1. Four bells commence tolling together. They toll at intervals of 1, 1 ,1 ,1 seconds respectively.
4 2 4
After what intervals will they toll together again?
(a) 1 min 30 sec. (b) 1 min 45 sec.
(c) 1 min 12 sec. (d) none of these

The unit digit of  6717 


103
2. is

(a) 3 (b) 7
(c) 9 (d) 2

The right most non-zero digit of  80 


1365
3. is

(a) 4 (b) 2
(c) 8 (d) 6
4. Suppose a and b are positive integers neither of which is a multiple of 3 then the remainder when
a 2  b 2 is divided by 3

(a) must be 0 (b) must be 1


(c) must be 2 (d) may be 1 or 2 but not 0

www.mstclasses.com, www.satpinnacle.com +91-9140474462, +91-8106429979 |46


Std. 10th |MATHEMATICS

5. A boy writes all the numbers from 100 to 999. The number of zeros that he uses is ‘a’ the number of
5’s that he uses is ‘b’ and the number of 8’s he uses is ‘c’. What is the value of b  c  a ?
(a) 280 (b) 380
(c) 180 (d) 80
6. The sum of two co-prime numbers is 43 and their LCM is 450, which of the following are numbers
(a) 18 (b) 30
(c) 25 (d) 15
7. Let n be the first 2006 positive integers are divisible by all of the numbers 1, 2, 3, 4, 5 and 7 then n
will be divisible by
(a) 1 (b) 2
(c) 3 (d) 4

8. 86  56 is divisible by

(a) 91 (b) 49
(c) 129 (d) 13

www.mstclasses.com, www.satpinnacle.com +91-9140474462, +91-8106429979 |47


Std. 10th |MATHEMATICS

Answer keys
Chapter Practice Problems
1. 2 2. 138
4. 64 5. 0
6. 16 8. 51
9. 18 10. 40
ASSIGNMENT
SUBJECTIVE
Section - A
2. 720 4. 2  72

5. 22  33  52

6. q has to be of the form 2m  5n where m and n are non-negative integers.


7. (i), (ii)
8. Terminating
10. (a) Irrational (b) Irrational (c) Irrational
(d) Irrational (e) Irrational (f) Rational
11. non-terminating 12. Terminating
13. Yes 14. No
15. No 16. 22
17. 5187 18. 22338
19. 12 and 144 21. 35
22. 4 23. Not of the form 2m × 5n
25. 301 27. 17
29. –19

www.mstclasses.com, www.satpinnacle.com +91-9140474462, +91-8106429979 |48


Std. 10th |MATHEMATICS

Section - B

1. 5 5
2. (a) 33  5  7 2 (b) 22  33  7 (c) 52  7 4
3. Yes. 4. 364
5. 3 6. 130
7. 2 8. 2520
9. 96 10. A, B
12. 15 hrs. 17. 0
18. HCF = 32, LCM = 128 19. 360
23. 3/1400 26. 16
27. 64 28. 63
29. 5 times 30. 255, 195
31. 44 hrs. 33. 95
34. No, because an integer can be written in the form 4q, 4q + 1, 4q + 2, 4q + 3.
35. True, because n(n + 1) will always be even, as one out of n or (n + 1) must be even.
36. True, because n(n + 1) (n + 2) will always be divisible by 6, as atleast one of the factors will be
divisible by 2 and atleast one of the factors will be divisible by 3.
37. No. Since positive integer can be written as 3q, 3q + 1, 3q + 2, therefore, square will be
9q 2  3m, 9q 2  6q  1  3  3q 2  2q   1  3m  1, 9q 2  12q  3  1  3m  1 .

No.  3q  1  9q 2  6q  1  3  3q 2  2q   3m  1 .
2
38.

39. HCF = 75, as HCF is the highest common factor.


40. 3  5  7  7  7  3  5  1  7 (16), which has more than two factors.

41. No, because HCF (18) does not divide LCM (380).
987 47
42. Terminating decimal expansion, because and 500 = 53 2 2
10500 500
 987 329 329 47 
10500  3500  22  53  7  2253  .094

43. Since 327.7081 is a terminating decimal number, so q must be of the form 2m  5n ; m, n are natural
numbers.
51. 63 52. 625

www.mstclasses.com, www.satpinnacle.com +91-9140474462, +91-8106429979 |49


Std. 10th |MATHEMATICS

55. 2520 cm 56. 23  54 , 0.0514


OBJECTIVE
Level – I
Multiple Choice Questions (Single Option Correct)
1. A 2. A 3. A 4. C
5. A 6. C 7. C 8. A
9. C 10. A 11. D 12. B
13. D 14. A 15. C 16. C
17. D 18. C 19. B 20. A
21. B 22. C 23. A 24. D
25. D
(Match the following)
1. (A) –(iii) (B) – (i) (C)-(ii) (D) – (iv)
2. (A) – (iii) (B) – (i) (C) – (iv) (D) – (ii)
Level – II
Multiple Choice Questions (One or More than One option correct)
1. B 2. A 3. C 4. C
5. B 6. A, C 7. B, D 8. A, B, C, D

www.mstclasses.com, www.satpinnacle.com +91-9140474462, +91-8106429979 |50

You might also like